What does the force between electric charges depend on

Answers

Answer 1

Answer:

Electric force and the inverse square law

Both forces act along the line joining the objects (masses or charges). Both forces are inversely proportional to the square of the distance between the objects, this is known as the inverse-square law.


Related Questions

How powerful is an engine that can do 400 J of work in 10 seconds?
(Provide your answer in both "Watts" and "horsepower".)

Answers

Answer:

[tex]P=40 \ W[/tex]

Conceptual:

What is work?Work is simply the transfer of work over a displacement. Work is a Newton-meter which is called a Joule, J. Work can be calculated using the following formulas.

[tex]\boxed{\left\begin{array}{ccc}\text{\underline{Equations for Work:}}\\W=F \Delta rcos(\theta) \ \text{(Constant} \ \vec F) \\W= \int\limits^{r_2}_{r_1} {Fcos(\theta)} \, dr \ \text{(Varible} \ \vec F) \end{array}\right }[/tex]

The angle "θ" is the angle between the force applied and the direction of displacement.

What is power?Power is the amount work done per second, which is a J/s, and this is clumped together to create a Watt, W. Power can be calculated using the following formula.

[tex]\boxed{\left\begin{array}{ccc}\text{\underline{Formula for Power:}}\\\\P=\frac{W}{t} \end{array}\right}[/tex]

Explanation:

Given that an engine does 400 J of work in 10 seconds. Find the power of the engine.

[tex]W=400 \ J\\t=10 \ s[/tex]

Plug these values into the formula for power.

[tex]P=\frac{W}{t} \\\\\Longrightarrow P=\frac{400}{10}\\\\\therefore \boxed{P=40 \ W}[/tex]

Thus, the engines power is calculated.

A metal ring is dropped into a localized region of constant magnetic field, as indicated in the figure (Figure 1) . The magnetic field is zero above and below the region where it is finite. For each of the three indicated locations (1, 2, and 3), is the magnetic force exerted on the ring upward, downward, or zero? Where would each of ther numbers (1, 2, and 3) be placed if given the bins upward, downward, and zero?

Answers

For each of the three locations, the magnetic forces exerted on the ring are as follows:
- Location 1: Upward
- Location 2: Zero
- Location 3: Upward

In a localized region of constant magnetic field, when a metal ring is dropped, the magnetic force exerted on the ring depends on its position within the field. Let's consider the three indicated locations (1, 2, and 3):
1. When the ring is partially inside the magnetic field (location 1), there will be a change in the magnetic flux through the ring, which induces an electric current in the ring according to Faraday's law. This current, in turn, generates its own magnetic field, which opposes the original magnetic field. As a result, the magnetic force exerted on the ring at this position will be upward.
2. When the ring is completely inside the magnetic field (location 2), the magnetic flux through the ring remains constant. Since there is no change in the magnetic flux, there is no induced electric current, and consequently, no magnetic force acting on the ring. The magnetic force at this position is zero.
3. When the ring is partially outside the magnetic field (location 3), similar to location 1, there will be a change in the magnetic flux through the ring, inducing an electric current. The generated magnetic field will again oppose the original field, creating an upward magnetic force on the ring.
In conclusion, for each of the three locations, the magnetic forces exerted on the ring are as follows:
- Location 1: Upward
- Location 2: Zero
- Location 3: Upward

To know more about Magnetic Forces visit:

https://brainly.com/question/31748676

#SPJ11

two loudspeakers in a 20°c room emit 686hz sound waves along the x- axis. an observer is located at x0.a. if the speakers are in phase, what is the smallest distance between the speakers for which the interference of the sound waves is perfectly destructive?b. if the speakers are out of phase, what is the smallest distance between the speakers for which the interference of the sound waves is maximum constructive?

Answers

Sure! Sound waves are vibrations that propagate through a medium, such as air, and can be described by their frequency, which is measured in hertz (Hz). Interference occurs when two or more waves overlap in space and time. If the waves are in phase, meaning their peaks and troughs align, they will create constructive interference, where the amplitude of the resulting wave is increased. If they are out of phase, meaning their peaks and troughs are misaligned, they will create destructive interference, where the amplitude of the resulting wave is decreased.

a. For destructive interference, we want the waves from the two speakers to cancel each other out. This occurs when the path difference between the waves is equal to a half-wavelength, or λ/2. The formula for wavelength is λ = v/f, where v is the speed of sound (343 m/s at 20°C) and f is the frequency (686 Hz). Therefore, λ = 343/686 = 0.5 m. The path difference between the waves at point x0 will depend on the distance between the speakers, which we'll call d. If d is the smallest distance for which we get destructive interference, then the path difference will be λ/2. Using the geometry of the situation, we can see that this occurs when sinθ = λ/(2d), where θ is the angle between the line connecting the speakers and the observer and the x-axis. Since θ = 10° (half of the 20° angle between the x-axis and the line connecting the speakers), we can solve for d: d = λ/(2sinθ) = 0.086 m.

b. For constructive interference, we want the waves from the two speakers to reinforce each other. This occurs when the path difference between the waves is equal to an integer number of wavelengths, or nλ. If the speakers are out of phase, the path difference will be λ/2 + nλ, where n is an odd integer. If the speakers are in phase, the path difference will be nλ, where n is an even integer. In either case, we want the path difference to be as small as possible, which means n should be as small as possible. Since we want constructive interference, we'll choose the smallest even integer, which is n = 2. Therefore, the path difference is 2λ = 1 m. Using the same formula as before, sinθ = nλ/(2d), we can solve for d: d = nλ/(2sinθ) = 0.214 m.

learn more about interference

https://brainly.in/question/13178543?referrer=searchResults

#SPJ11

Sam is stationary and then starts skateboarding. His velocity increases to 5m/s west over a period of 10 seconds. What is Sam’s average acceleration?

Answers

Sam's average acceleration is 0.5 m/s² west, calculated by dividing the change in velocity (5 m/s) by the time (10 s).

Sam is initially stationary and then starts skateboarding with his velocity increasing to 5 meters per second (m/s) west over a period of 10 seconds.

To find his average acceleration, we need to divide the change in velocity by the time it took for the change to occur.

In this case, Sam's change in velocity is 5 m/s (from 0 m/s to 5 m/s) and the time taken is 10 seconds.

By dividing the change in velocity (5 m/s) by the time (10 s), we find that Sam's average acceleration is 0.5 meters per second squared (m/s²) west.

For more such questions on velocity, click on:

https://brainly.com/question/80295

#SPJ11

true/false. the ideal estimator has the greatest variance among all unbiased estimators.

Answers

The statement: the ideal estimator has the greatest variance among all unbiased estimators is FALSE because the ideal estimator is the estimator with the minimum variance among all unbiased estimators.

This is known as the minimum variance unbiased estimator (MVUE) and is highly desirable in statistics. An estimator is said to be unbiased if its expected value is equal to the true value of the parameter being estimated.

The variance of an estimator measures how spread out its values are from its expected value, and a lower variance indicates a more precise estimator. Therefore, the MVUE is the estimator that achieves both unbiasedness and minimum variance simultaneously.

In some cases, the MVUE may not exist, or it may be difficult to find. However, if an MVUE exists, it is the best unbiased estimator in terms of precision.

To know more about variance, refer here:

https://brainly.com/question/29253308#

#SPJ11

If a sheet containing a single thin slit is heated (without damaging it) and therefore expands, what happens to the width of the central bright diffraction region on a distant screen? A.it gets narrower B.It gets wider C. It doesnt change

Answers

The width of the central bright diffraction region on a distant screen will get wider if a sheet containing a single thin slit is heated and expands. This is because the width of the central bright diffraction region is directly proportional to the width of the slit.  Correct answer is option B

As the slit expands due to heating, its width also increases, leading to a wider central bright diffraction region on the distant screen. This phenomenon can be explained by the principles of diffraction, which states that when a wave passes through an aperture, it diffracts and spreads out. In the case of a single thin slit, the light passing through the slit diffracts and creates a pattern of alternating bright and dark fringes on the distant screen.

The central bright fringe corresponds to the direct transmission of light through the center of the slit, and its width is dependent on the width of the slit.



Therefore, if the slit expands due to heating, the width of the central bright fringe also increases, resulting in a wider diffraction pattern on the screen. However, it is important to note that the intensity of the bright fringe decreases as the width increases, leading to a dimmer diffraction pattern overall.  Correct answer is option B

Know more about bright fringe here:

https://brainly.com/question/31315270

#SPJ11

A photon with a wavelength of 3.40×10−13 m strikes a deuteron, splitting it into a proton and a neutron.
A) Calculate the kinetic energy released in this interaction. (MeV)
B)Assuming the two particles share the energy equally, and taking their masses to be 1.00 u, calculate their speeds after the photodisintegration. (m/s)

Answers

The kinetic energy released in this interaction is approximately [tex]5.83 * 10^{-13} J[/tex], or 0.364 MeV, and the speeds of the proton and neutron after the photodisintegration are approximately [tex]2.84 * 10^6[/tex] m/s.

A) To calculate the kinetic energy released in this interaction, we need to find the initial and final energies of the photon and the deuteron, respectively, and then subtract them.

The initial energy of the photon can be found using the formula E = hc/λ, where h is Planck's constant, c is the speed of light, and λ is the wavelength of the photon.

E = hc/λ = [tex](6.626 * 10^{-34} Js * 2.998 * 10^8 m/s)/(3.40 * 10^{-13} m) = 5.83 * 10^{-13} J[/tex]

The final energies of the proton and neutron can be found using the formula E =[tex](1/2)mv^2[/tex], where m is the mass of each particle and v is their velocity.

Since the two particles share the energy equally, each will have an energy of [tex]5.83 * 10^{-13} J/2 = 2.92 * 10^{-13} J.[/tex]

The mass of a proton and a neutron is approximately 1.0073 u. Converting to kilograms, we get:

m = [tex]1.0073 u * 1.661 * 10^{-27} kg/u = 1.674 * 10^{-27} kg[/tex]

The kinetic energy of each particle is:

E = [tex](1/2)mv^2 = 2.92 * 10^{-13} J[/tex]

Solving for v, we get:

v = [tex]\sqrt{2E/m} = 2.84 * 10^6 m/s[/tex]

For more question on kinetic energy click on

https://brainly.com/question/8101588

#SPJ11

The wavelength of the photon is important because it determines its energy. The shorter the wavelength, the higher the energy. In this case, the photon has a relatively short wavelength, meaning it has a high amount of energy.

The concept of energy is crucial to understanding what happens after the photodisintegration. When the deuteron splits, the two resulting particles will share the energy equally. Since their masses are equal, this means they will also have equal speeds. By calculating the energy of the photon, we can determine the amount of energy that each particle receives and from there, we can calculate their speeds.

Finally, the term "photon" refers to a packet of energy that behaves like a particle. Photons are the building blocks of light and electromagnetic radiation. They have no mass, but they do have energy, which is directly proportional to their wavelength.
To learn more about photon click here:brainly.com/question/20912241  

#SPJ11

Two people of the same mass climb the same flight of stairs the first two person climbs the stairs in 25 seconds the second person does so in 35 seconds who does the greater work:
a. the 1st person since he accomplished the task first
b. the 2nd person since he did the work longer
c. both did the same amount of work
d. cannot be determeined​

Answers

(c) Both did the same amount of work. The work done is determined by the force exerted on the stairs multiplied by the distance traveled.

(c) Both did the same amount of work. The work done is determined by the force exerted on the stairs multiplied by the distance traveled. In this scenario, the force exerted by each person is their weight, which is directly proportional to their mass. Since both people have the same mass, their weight and force exerted on the stairs are equal. Additionally, since they climbed the same flight of stairs, the distance traveled is also the same for both individuals. Therefore, the work done by each person is equal. The time taken to complete the task does not affect the amount of work done, as work is independent of the duration of the activity.

Learn more about mass here:

https://brainly.com/question/11954533

#SPJ11

calculate the burnout velocity required to transfer the probe between the vicinity of the earth and the moon's orbit using a hohmann transfer

Answers

The burnout velocity required to transfer the probe between the vicinity of the earth and the moon's orbit using a Hohmann transfer is estimated to be 3.06 km/s.

To travel between two celestial bodies, such as Earth and the Moon, a spacecraft must follow a specific trajectory that requires a certain amount of energy. The Hohmann transfer is a commonly used method for transferring a spacecraft from one circular orbit to another by using a minimum amount of energy.

To calculate the burnout velocity required to transfer the probe between the vicinity of the Earth and the Moon's orbit using a Hohmann transfer, we can use the following formula:

V = √(μ(2/r₁ - 1/a) - μ(2/r₂ - 1/a))

Where V is the burnout velocity, μ is the gravitational parameter (3.986 × 10⁵ km³/s² for Earth), r₁ is the initial radius (Earth's radius + altitude), r₂ is the final radius (Moon's radius + altitude), and a is the semi-major axis of the transfer ellipse.

Assuming the altitude of the initial orbit is 200 km above Earth's surface and the altitude of the final orbit is 100 km above the Moon's surface, we can calculate the required burnout velocity using the above formula. The semi-major axis of the transfer ellipse can be found using the following formula:

a = (r₁ + r₂) / 2

Substituting the values and solving the equations, we get the burnout velocity to be approximately 3.06 km/s.

To know more about the Hohmann, here

https://brainly.com/question/13389845

#SPJ4

Assume that the focus is at the pole, the major axis lies on the polar axis, and the length of the major axis is. show that the polar equation of the orbit of a planet is where is the eccentricity.

Answers

The polar equation of the orbit of a planet with a focus at the pole, major axis on the polar axis, and length of major axis a is r = a(1-e^2)/(1+e*cos(theta)).

When a planet orbits a star, the shape of the orbit can be described using a polar equation. In this case, the focus is at the pole, which means that the planet's distance from the star is the same as the distance from the pole. The major axis lies on the polar axis, which means that the distance between the star and the farthest point on the orbit is a. Finally, the length of the major axis is related to the eccentricity of the orbit, which is the ratio of the distance between the foci to the length of the major axis.

Using these parameters, we can derive the polar equation of the orbit as r = a(1-e^2)/(1+e*cos(theta)), where r is the distance from the star to the planet, theta is the angle from the polar axis, a is the length of the major axis, and e is the eccentricity.

Learn more about eccentricity here:

https://brainly.com/question/30653946

#SPJ11

Consider a meter stick that oscillates back and forth about a pivot point at one of its ends.
Is the period of a simple pendulum of length L=1.00m greater than, less than, or the same as the period of the meterstick?

Answers

The period of a simple pendulum of length L=1.00m is less than the period of the meter stick oscillation.

The period of this oscillation can be calculated using the formula T = 2π√(I/mgd), where T is the period, I is the moment of inertia of the meter stick, m is its mass, g is the acceleration due to gravity, and d is the distance from the pivot point to the center of mass of the meter stick.

Now, if we compare this with the period of a simple pendulum of length L = 1.00m, which can be calculated using the formula T = 2π√(L/g), we see that the period of the pendulum depends only on its length and the acceleration due to gravity. Therefore, we can conclude that the period of the meter stick oscillation is not the same as that of the simple pendulum.

In fact, since the meter stick is much longer than the simple pendulum, its moment of inertia and distance from the pivot point are much larger. This results in a longer period of oscillation for the meter stick compared to the pendulum. Therefore, we can say that the period of a simple pendulum of length L=1.00m is less than the period of the meter stick oscillation.

To know more about the moment of inertia, click here;

https://brainly.com/question/15246709

#SPJ11

which group of elements has a full octet of electrons

Answers

The group of elements that has a full octet of electrons is the noble gases.

The noble gases, also known as the inert gases, are the elements found in group 18 of the periodic table. This group includes helium, neon, argon, krypton, xenon, and radon.

These elements have a complete valence shell of electrons, which means that their outermost energy level is fully occupied with eight electrons, except for helium, which has only two electrons in its outermost energy level. This makes noble gases highly stable and unreactive, as they do not have a tendency to gain or lose electrons to form chemical bonds with other elements.

In summary, the noble gases have a full octet of electrons, which makes them highly stable and unreactive. This property is due to the complete valence shell of electrons in their outermost energy level.

To know more about electrons, visit;

https://brainly.com/question/860094

#SPJ11

A particle moves along the x-axis so that its velocity at time t is given by v(t) = ((t^6)-(13t^4)+(12)) / (10t^3)+3 At time t = 0, the initial position of the particle is x = 7. Find the acceleration of the particle at time t = 5.1.

Answers

A particle's velocity is given by v(t) = ((t⁶)-(13t⁴)+(12)) / (10t³)+3, and its initial position is x=7 at time t=0. The acceleration of the particle at time t=5.1 is approximately -7.8 m/s².

The first step is to find the acceleration of the particle, which can be obtained by taking the derivative of the velocity function v(t) with respect to time t. Thus, we have:

a(t) = v'(t) = ((6t⁵)-(52t³)) / ((10t³)+3)²

To find the acceleration of the particle at time t = 5.1, we substitute t = 5.1 into the acceleration function to get:

a(5.1) = ((6(5.1)⁵)-(52(5.1)³)) / ((10(5.1)³)+3)²

Simplifying this expression, we get:

a(5.1) ≈ -7.8 m/s²

Therefore, the acceleration of the particle at time t = 5.1 is approximately -7.8 m/s².

To know more about the velocity refer here :

https://brainly.com/question/2136991#

#SPJ11

a 11.3 cm long solenoid contains 895 turns and carries a current of 5.93 a . what is the strength of the magnetic field at the center of this solen

Answers

The strength of the magnetic field at the center of the solenoid is 1.87 millitesla (mT), or 1.87 x 10^-3 T.

To calculate the strength of the magnetic field at the center of the 11.3 cm long solenoid with 895 turns and a current of 5.93 A, we can use the formula:
B = μ₀ * n * I
where B is the magnetic field strength, μ₀ is the permeability of free space (4π x 10^-7 Tm/A), n is the number of turns per unit length, and I is the current.
First, we need to find the number of turns per unit length (n) by dividing the total number of turns (895) by the length of the solenoid (11.3 cm or 0.113 m):
n = 895 / 0.113 = 7921 turns/m
Now we can plug in the values and solve for B:
B = (4π x 10^-7 Tm/A) * 7921 turns/m * 5.93 A
B = 1.87 x 10^-3 T
Therefore, the strength of the magnetic field at the center of the solenoid is 1.87 millitesla (mT), or 1.87 x 10^-3 T. This value is directly proportional to the current flowing through the solenoid and the number of turns per unit length.

To know more about solenoid visit:

https://brainly.com/question/15504705

#SPJ11

Consider an X-ray tube with an applied voltage of 44 kV Part (a) What is the photon energy, in keV, of the shortest-wavelength X-ray radiation that can be generated? Part (b) What is the wavelength, in meters, of that radiation?

Answers

The photon energy of the shortest-wavelength X-ray radiation that can be generated with an applied voltage of 44 kV is 44 keV. The wavelength of the radiation is 1.79 x 10^-11 meters.

We are given an applied voltage of 44 kV and we are asked to find the photon energy and wavelength of the shortest-wavelength X-ray radiation that can be generated.

To do this, we need to use the relationship between the energy of a photon and the applied voltage of the tube, which is given by the formula E = hc/λ = eV, where E is the energy of the photon, h is Planck's constant, c is the speed of light, λ is the wavelength of the photon, and e is the electronic charge.

Using this formula, we can calculate the energy of the photons generated by the tube at an applied voltage of 44 kV, which turns out to be 44 keV. We can then use this energy to calculate the wavelength of the photons using the formula λ = hc/E.

The resulting wavelength is 1.79 x 10^-11 meters, which is the shortest-wavelength X-ray radiation that can be generated by the tube at this voltage.

To know more about wavelength, refer here:

https://brainly.com/question/13047641#

#SPJ11

An EM wave has frequency 8.59×10 14
Hz. Part A What is its wavelength? * Incorrect; Try Again; 2 attempts remaining Part B How would we classity it? infrared visible light

Answers

Part A: The wavelength of an EM wave with a frequency of 8.59×10^14 Hz is approximately 3.49×10^-7 meters.

Part B: This EM wave would be classified as visible light.

To determine the wavelength of an electromagnetic (EM) wave, you can use the formula: wavelength = speed of light / frequency. The speed of light is approximately 3.00×10^8 meters per second. Using the given frequency of 8.59×10^14 Hz, the wavelength can be calculated as follows:

Wavelength = (3.00×10^8 m/s) / (8.59×10^14 Hz) ≈ 3.49×10^-7 meters

As for the classification, the electromagnetic spectrum is divided into different regions based on wavelength or frequency. Visible light has wavelengths ranging from approximately 4.00×10^-7 meters (400 nm) to 7.00×10^-7 meters (700 nm). Since the calculated wavelength of this EM wave (3.49×10^-7 meters) falls within this range, it would be classified as visible light.

To know more about electromagnetic wave, click here;

https://brainly.com/question/3101711

#SPJ11

In what direction is matter displaced in a traverse wave

A. In the same direction of the energy flow

B. In the opposite direction of the energy flow

C. In a spiral around the energy flow

D. At a right angle to the energy flow

Answers

Answer:

In a transverse wave, the matter is displaced perpendicular (at a right angle) to the direction of energy flow. Therefore, the correct answer is D.

Explanation:

The amount of energy released when 45g of -175 degrees C steam is cooled to 90 degrees C is
419481
317781
101700
417600

Answers

The energy released during the cooling of 45g of -175 degrees C steam to 90 degrees is 419,481,317,781,101,700,417,600 J

When steam at -175 degrees Celsius is cooled to 90 degrees Celsius, it undergoes a phase change from a gas to a liquid. During this process, energy is released in the form of heat as the molecules of the steam lose kinetic energy and transition to a lower energy state. This energy release is due to the formation of intermolecular forces between the water molecules, which creates a more stable state of matter.

To calculate the amount of energy released, we can use the formula Q = mΔTc, where Q is the energy released, m is the mass of the steam, ΔT is the change in temperature, and c is the specific heat capacity of water. Since the steam undergoes a phase change, we need to also include the energy required for this process, which is known as the latent heat of vaporization.

The energy released during the cooling of 45g of -175 degrees C steam to 90 degrees C can be calculated as follows:

Q = (45g) * (90°C - (-175°C)) * (4.184 J/(g·°C)) + (45g) * (2257 J/g)

= 419,481,317,781,101,700,417,600 J

This calculation shows that an enormous amount of energy is released during the cooling of steam, due to the large latent heat of vaporization of water. This energy can be harnessed and used for various purposes, such as generating electricity in power plants or powering steam engines.

To know more about energy released, refer to the link below:

https://brainly.com/question/21592984#

#SPJ11

The heat exchanger in problem 1 is a parallel-flow concentric tube heat exchanger. Hint: note the temperature changes of cold and hot fluids. True or False

Answers

True

The statement suggests that in problem 1, there are temperature changes in both the hot and cold fluids that flow through a parallel-flow concentric tube heat exchanger.

To know more about True refer here

https://brainly.com/question/30615879#

#SPJ11

a sinusoidal electromagnetic wave has rms electric field 800 n/c. what is the intensity of the wave? (c = 3.00 × 108 m/s, μ0 = 4π × 10-7 t ∙ m/a, ε0 = 8.85 × 10-12 c2/n ∙ m2)

Answers

The intensity of the wave is 1.92 x [tex]10^{-5[/tex] W/[tex]m^2[/tex].

The intensity (I) of an electromagnetic wave is defined as the average power per unit area that it carries.

The relationship between the intensity and the RMS electric field (E) of a sinusoidal electromagnetic wave is given by:

I = (1/2) x [tex]\epsilon_0[/tex] x c x [tex]E^2[/tex]

where [tex]\epsilon_0[/tex] is the vacuum permittivity and c is the speed of light in vacuum.

Substituting the given values, we have:

I = [tex](1/2) \times (8.85 \times 10^{-12}) \times (3.00 \times 10^8) \times (800 \times 10^{-9})^2[/tex]

I = 9.44 × [tex]10^{-5} W/m^2[/tex]

Therefore, the intensity of the wave is 9.44 × [tex]10^{-5[/tex] W/[tex]m^2[/tex]

It is important to note that the intensity of an electromagnetic wave depends on the square of the amplitude of its electric field.

Therefore, doubling the RMS electric field of the wave will result in a four-fold increase in its intensity.

Conversely, reducing the electric field amplitude by a factor of 2 will result in a reduction of the wave's intensity by a factor of 4.

For similar question on intensity of the wave

https://brainly.com/question/24319848

#SPJ11

The intensity (I) of an electromagnetic wave is defined as the average power (P) per unit area (A) of the wave, and can be calculated using the following formula:

I = P/A

We can also express the power of an electromagnetic wave in terms of the electric field (E) and magnetic field (B) amplitudes, using the following relationship:

P = (1/2)ε0cE^2

where ε0 is the permittivity of free space, c is the speed of light, and E is the root-mean-square (rms) electric field amplitude.

Since we are given the rms electric field amplitude, we can use the above equation to calculate the power of the wave:

P = (1/2)ε0cE^2 = (1/2)(8.85 × 10^-12 c^2/n ∙ m^2)(3.00 × 10^8 m/s)(800 × 10^-9 V/m)^2 = 0.095 W/m^2

Next, we can calculate the intensity by dividing the power by the area through which the wave is passing. Since the area of a sphere of radius r is 4πr^2, and the wave is assumed to be spreading out uniformly in all directions, we can take the area to be that of a sphere with radius r = 1 meter:

A = 4πr^2 = 4π(1 m)^2 = 12.57 m^2

Therefore, the intensity of the wave is:

I = P/A = 0.095 W/m^2 ÷ 12.57 m^2 = 7.57 × 10^-3 W/m^2

So the intensity of the wave is 7.57 × 10^-3 W/m^2.

Learn more about electromagnetic wave here : brainly.com/question/29774932

#SPJ11

FILL IN THE BLANK ____ satellites travel at a speed and direction that keeps pace with the earth's rotation, so they appear (from earth) to remain stationary over a given spot.

Answers

Answer:

Geosynchronous

Explanation:

Quizlet

Geostationary satellites travel at a speed and direction that keeps pace with the earth's rotation, so they appear (from earth) to remain stationary over a given spot.

Tall Pacific Coast redwood trees (Sequoia sempervirens) can reach heights of about 100 m. If air drag is negligibly small, how fast is a sequoia cone moving when it reaches the ground if it dropped from the top of a 100 m tree?

Answers

To determine the speed at which a sequoia cone would hit the ground when dropped from the top of a 100 m tall tree, we can use the principles of free fall motion.

When air drag is negligible, the only force acting on the cone is gravity. The acceleration due to gravity, denoted as "g," is approximately 9.8 m/s² on Earth.

The speed (v) of an object in free fall can be calculated using the equation:

v = √(2gh),

where h is the height from which the object falls. In this case, h is 100 m.

Plugging in the values:

v = √(2 * 9.8 m/s² * 100 m) ≈ √(1960) ≈ 44.27 m/s.

Therefore, the sequoia cone would be moving at approximately 44.27 meters per second (m/s) when it reaches the ground.

To know more about sequoia cone refer here

https://brainly.com/question/36336250#

#SPJ11

A girl strikes a 0.445kg soccer ball with a net force of 5.92N. What is the acceleration of the soccer ball? 0 13.3 m/s2 O 0.0752 m/s2 0 6.36 m/s2 0 5.48 m/s2

Answers

The answer to the question is 13.3 m/s2, if a girl strikes a 0.445kg soccer ball with a net force of 5.92N.


To find the acceleration of the soccer ball, we can use the formula F = ma, where F is the net force applied to the ball, m is the mass of the ball, and a is the acceleration of the ball. We know that the mass of the ball is 0.445kg and the net force applied is 5.92N. Substituting these values into the formula, we get:

5.92N = 0.445kg x a

Solving for a, we get:

a = 5.92N / 0.445kg

a ≈ 13.3 m/s2

Therefore, the answer is that the acceleration of the soccer ball is 13.3 m/s2.

To learn more about acceleration visit:
brainly.com/question/2303856

#SPJ11

Consider an infinite parallel-plate capacitor, with the lower plate (at z = - d / 2 ) carrying surface charge density -o, and the upper plate (at z = d / 2 ) carrying charge density +o.
(a) Determine all nine elements of the stress tensor, in the region between the plates. Display your answer as a 3 * 3 matrix:
(b) Use Eq. 8.21 to determine the electromagnetic force per unit area on the top plate. Compare Eq. 2.51.
(c) What is the electromagnetic momentum per unit area, per unit time, crossing the xy plane (or any other plane parallel to that one, between the plates)?
(d) Of course, there must be mechanical forces holding the plates apart-perhaps the capacitor is filled with insulating material under pressure. Suppose we sud- denly remove the insulator; the momentum flux (c) is now absorbed by the plates, and they begin to move. Find the momentum per unit time delivered to the top plate (which is to say, the force acting on it) and compare your answer to (b). [Note: This is not an additional force, but rather an alternative way of calculating the same force-in (b) we got it from the force law, and in (d) we do it by conservation of momentum.]

Answers

(a) The stress tensor elements for an infinite parallel-plate capacitor in the region between the plates are:

Txx = Tyy = (ε/2)E²Tzz = -TxxTxy = Tyx = Txz = Tzx = Tzy = Tyz = 0

(b) Using Eq, the electromagnetic force per unit area on the top plate is:

F = ε/2 * E² = Tzz

Comparing with Eq. 2.51, the electromagnetic force per unit area is equal to the energy density per unit volume.

(c) The electromagnetic momentum per unit area, per unit time, crossing the xy plane (or any other plane parallel to that one, between the plates) is zero, as there is no magnetic field in this region.

(d) The momentum per unit time delivered to the top plate when the insulator is removed is also equal to Tzz, which is the force acting on the top plate.

(b) The element responsible for the pressure on the top plate is Tzz, which is negative and equal in magnitude to Txx and Tyy, indicating that there is a compressive force acting in the z-direction.

(a) The stress tensor is a 3x3 matrix that describes the stress and strain in a material. For an infinite parallel-plate capacitor in the region between the plates, the stress tensor has the following elements:

Txx = Tyy = (ε/2)E², which represents the pressure acting in the x and y directions due to the electric field between the plates.

Tzz = -Txx, which represents the compressive force acting in the z-direction due to the pressure difference between the plates.

Txy = Tyx = Txz = Tzx = Tzy = Tyz = 0, which indicates that there are no shear forces acting between the plates.

(b) The electromagnetic force per unit area on the top plate is given by the negative of the diagonal element Tzz of the stress tensor, which is equal to ε/2 * E². This is in agreement with Eq. 2.51, which states that the electromagnetic force per unit area is equal to the energy density per unit volume.

(c) There is no magnetic field between the plates, so the electromagnetic momentum per unit area, per unit time, crossing any plane parallel to the plates is zero.

(d) The momentum per unit time delivered to the top plate when the insulator is removed is equal to Tzz, which is the force acting on the top plate. This is consistent with the result obtained in part (b), which shows that the electromagnetic force per unit area on the top plate is also equal to Tzz.

(b) The element responsible for the pressure on the top plate is Tzz, which is negative and equal in magnitude to Txx and Tyy. This indicates that there is a compressive force acting in the z-direction due to the pressure difference between the plates.

To learn more about electromagnetic momentum, here

https://brainly.com/question/14214744

#SPJ4

ASAP HELP PLEASE

The speed of light is 300,000,000 m/s, the speed of sound is 343 m/s.


If an airplane is 10 km (10,000 m), how much time difference would there be between you seeing the plane and hearing it?


DO NOT ABUSE THE POINT SYSTEM

YOU WILL BE REPORTED

Answers

The time difference between seeing the airplane and hearing it would be:

Time difference = time for light to travel - time for sound to travel

= 0.0000333 s - 29.15 s = -29.15 seconds (approx.)

This negative time difference means that we would hear the sound of the airplane before we see it, since the sound takes longer to reach us than the light.

To calculate the time difference between seeing an airplane and hearing it, we need to determine how long it takes for the sound to travel from the airplane to our ears. We can then subtract this time from the time it takes for the light to travel from the airplane to our eyes.

The distance between us and the airplane is 10,000 meters. Since sound travels at a speed of 343 m/s, we can divide the distance by the speed of sound to get the time it takes for the sound to reach us:

Time for sound to travel = distance / speed of sound = 10,000 m / 343 m/s = 29.15 seconds (approx.)

On the other hand, since light travels at a speed of 300,000,000 m/s, we can divide the distance by the speed of light to get the time it takes for the light to reach us:

Time for light to travel = distance / speed of light = 10,000 m / 300,000,000 m/s = 0.0000333 seconds (approx.)

Therefore, the time difference between seeing the airplane and hearing it would be:

Time difference = time for light to travel - time for sound to travel

= 0.0000333 s - 29.15 s = -29.15 seconds (approx.)

This negative time difference means that we would hear the sound of the airplane before we see it, since the sound takes longer to reach us than the light.

For more question on time difference

https://brainly.com/question/20352909

#SPJ11

Consider a table that measures 1.6mx2.6m. The atmospheric pressure is 1.0x105 Determine the magnitude of the total force of the atmosphere acting on the top of the table. a) 1.96x10°N b) 4.16x10³N c) 1.96x105 N d) 4.96x10 N e) 4.16x10* N

Answers

The magnitude of the total force of the atmosphere acting on the top of the table with dimensions 1.6m x 2.6m and atmospheric pressure is 1.0 x 10^5 is 4.16 x 10^5 N. Therefore, the correct option is E.

To determine the magnitude of the total force of the atmosphere acting on the top of the table, you can use the formula:

Force = Pressure × Area.

Given the dimensions of the table (1.6m x 2.6m) and the atmospheric pressure (1.0 x 10^5 Pa), you can calculate the force as follows:

Force = (1.0 x 10^5 Pa) × (1.6 m × 2.6 m)

Force = (1.0 x 10^5 Pa) × (4.16 m²)

Force = 4.16 x 10^5 N

Thus, the magnitude of the total force of the atmosphere acting on the top of the table is 4.16 x 10^5 N which corresponds to option E.

Note: The question is incomplete. The complete question probably is: Consider a table that measures 1.6mx2.6m. The atmospheric pressure is 1.0x10^5. Determine the magnitude of the total force of the atmosphere acting on the top of the table. a) 1.96x10°N b) 4.16x10³N c) 1.96x10^5 N d) 4.96x10 N e) 4.16x10^5 N.

Learn more about Force:

https://brainly.com/question/29875769

#SPJ11

The coefficients of friction between the 20 kg crate and the inclined surface are µs = 0.24 and µk = 0.22. If the crate starts from rest and the horizontal force F = 200 N. Determine if the Force move the crate when it start from rest. ENTER the value of the sum of Forces opposed to the desired movement

Answers

The force F is sufficient to move the crate when it starts from rest, and the sum of the forces opposed to the desired movement is 43.16 N.

When a force is applied to a crate on an inclined surface, the force required to start the movement is dependent on the coefficient of static friction (µ(s)) and the normal force (F(n)) acting on the crate. Once the crate starts moving, the force required to maintain the motion is dependent on the coefficient of kinetic friction (µ(k)) and the normal force.

In this problem, the coefficient of static friction and the coefficient of kinetic friction are given as µ(s) = 0.24 and µ(k) = 0.22, respectively. The force applied to the crate is F = 200 N, and the crate has a mass of 20 kg.

To determine if the force F can move the crate when it starts from rest, we need to calculate the maximum force of static friction Fs(max) that can act on the crate. This is given by:

F(s)(max) = µ(s) * F(n)

The normal force F(n) acting on the crate is equal to the weight of the crate, which is:

F(n) = mg

where m is the mass of the crate and g is the acceleration due to gravity (9.81 m/s^2). Substituting the values, we get:

F(n) = (20 kg) * (9.81 m/s²) = 196.2 N

Therefore, the maximum force of static friction is:

F(s)(max) = (0.24) * (196.2 N) = 47.09 N

Since the applied force F = 200 N is greater than the maximum force of static friction F(s)(max), the crate will move. The force that opposes the desired movement is the force of kinetic friction F(k), which is given by:

F(k) = µ(k) * F(n)

Substituting the values, we get:

F(k) = (0.22) * (196.2 N) = 43.16 N

Therefore, the sum of the forces opposed to the desired movement is:

F(sum) = F(k) = 43.16 N

Thus, the force F is sufficient to move the crate when it starts from rest, and the sum of the forces opposed to the desired movement is 43.16 N.

Learn more about friction at: https://brainly.com/question/31226

#SPJ11

the earth is approximately spherical, with a diameter of 1.27×107m1.27×107m. it takes 24.0 hours for the earth to complete one revolution.

Answers

Answer:This statement seems incomplete. Please provide the rest of the question.

learn more about  revolution.

https://brainly.com/question/17773408?referrer=searchResults

#SPJ11

four objects are situated along the y axis as follows: a 1.92-kg object is at 3.01 m, a 2.93-kg object is at 2.42 m, a 2.53-kg object is at the origin, and a 4.05-kg object is at -0.498 m. where is the center of mass of these objects?

Answers

The center of mass of the four objects is approximately 0.95 meters along the y-axis.

To find the center of mass (COM) of the four objects along the y-axis, we will use the formula:

COM = (m1*y1 + m2*y2 + m3*y3 + m4*y4) / (m1 + m2 + m3 + m4)

Where m1, m2, m3, and m4 are the masses of the objects, and y1, y2, y3, and y4 are their respective positions on the y-axis. Plugging in the given values:

COM = ((1.92 kg * 3.01 m) + (2.93 kg * 2.42 m) + (2.53 kg * 0 m) + (4.05 kg * -0.498 m)) / (1.92 kg + 2.93 kg + 2.53 kg + 4.05 kg)

COM = ((5.7792 kg*m) + (7.0936 kg*m) + (0 kg*m) + (-2.0169 kg*m)) / (11.43 kg)

COM = (10.8559 kg*m) / (11.43 kg)

COM ≈ 0.95 m

Learn more about center of mass here:-

https://brainly.com/question/27549055

#SPJ11

A gold wire with a circular cross-section has a mass of 1.10 g and a resistance of 0.720 Ω. At 20°C, the resistivity of gold is 2.44 ✕ 10−8 Ω · m and its density is 19,300 kg/m3.
How long (in m) is the wire?
m
What is the diameter (in mm) of the wire?
mm

Answers

The diameter of the wire is 0.42 mm. The length of the wire is 1.07 m.

The resistance of the gold wire can be calculated using the formula:

R = (ρL) / A

V = m / ρ

V = 1.10 g / (19,300 kg/m³)

V = 5.70 ✕ [tex]10^{-8[/tex] m^3

Next, we can calculate the length of the wire:

L = (RA) / ρ

L = (0.720 Ω)(πd²/4) / (2.44 ✕ [tex]10^{-8[/tex] Ω · m)

L = (0.720 Ω)(πd²/4) / (2.44 ✕ [tex]10^{-8[/tex] Ω · m)

L = 7.41 ✕ [tex]10^{-3[/tex] d²

Substituting the value of V into the equation above gives:

7.41 ✕ [tex]10^{-3[/tex] d² = 5.70 ✕ [tex]10^{-8[/tex]

Solving for d, we get:

d = 0.42 mm

Finally, we can use the length equation to calculate the length of the wire:

L = 7.41 ✕ [tex]10^{-3[/tex] d²

L = 7.41 ✕ [tex]10^{-3[/tex] (0.42 mm)²

L = 1.07 m

Resistance refers to the opposition that occurs when current flows through a conductor. It is an inherent property of a material that opposes the flow of electricity. Resistance is measured in ohms and is represented by the symbol Ω. The resistance of a conductor depends on several factors such as the material, the length of the conductor, its cross-sectional area, and the temperature.

Resistance is an important concept in electrical circuits as it affects the flow of current and voltage across a circuit. A higher resistance means a lower current and a higher voltage drop across the circuit. In electronic devices, resistors are used to control the flow of current and limit the voltage. Different materials have different resistivity, which is a measure of their resistance to the flow of electricity. Materials such as copper, aluminum, and gold have low resistivity and are commonly used in electrical wiring. Resistance plays a crucial role in determining the efficiency and performance of electrical and electronic devices.

To learn more about Resistance visit here:

brainly.com/question/30669051

#SPJ4

Other Questions
use two pieces of data to comment on whether the ratio of (e,e)-1,4-diphenyl-1,3-butadiene to (e,z)-1,4-diphenyl-1,3-butadiene changed after recrystallization. The accepted value for Ag2CrO4 is 1.1 x 10 -12. Calculate the percent error in your experimentally determined value of Ksp: 4.9 x 10-10. ______ is the area of medicine that deals with the protection and improvement of citizen health and hygiene by government agencies. antonimo de ateniense which instrument family provides the majority of the harmony in this excerpt, from the a section, which begins about 47 seconds into the piece? On the moon, the acceleration due to gravity is 1.6m/sec^2If a rock is dropped into a crevasse, how fast will it be going just before it hits. bottom 30 sec later? (b) How far below the point of release is the bottom of the crevasse? (c) If instead of being released from rest, the rock is thrown into the crevasse from the same point with a downward velocity of 4 m/s, when will it hit the bottom and how fast will it be going when it does?How far below the point of release is the bottom of the crevasse? how would a decline in vo2 max affect step length and walking speed Which legal procedure is used to determine a fair price in an eminent domain action?A) CondemnationB) EscheatC) Ag valoremD) Eviction the red and blue pigments stored in vacuoles in flowers are: error propagation. a quantity of interest q is a function of x: q = (1 - x2) * cos( (x 2) / x3 ) given x = 1.70 0.02, calculate the uncertainty in q. round your answer to three (3) decimal places. in a study of cardiac invalidism (taylor et al., 1985), wives' perceptions of their husbands' cardiac and physical efficiency were highest when they had Jenna collected data modeling a company's company costs versus its profits. The data are shown in the table: x g(x) 2 2 1 3 0 2 1 17 Which of the following is a true statement for this function? The function is decreasing from x = 2 to x = 1. The function is decreasing from x = 1 to x = 0. The function is increasing from x = 0 to x = 1. The function is decreasing from x = 0 to x = 1. genetics may determine the age at which a woman begins the transition to menopause. True or False "Yes, it is true that he killed both his parents. Nevertheless, you shouldn't be too hard on him; after all, he is an orphan." His is an example of which kind of informal fallacy?Group of answer choicesA. Slippery slope.B. Common clause.C. Appeal to force.D. Appeal to pity. under the third prong (i.e., part) of the current test used by the u.s. supreme court for determining whether material is obscene, the fact finder must determine if the speech in question using hess's law, calculate h for the process: sb (s) cl2 (g) sbcl5 (s) from the following information: sb (s) cl2 (g) sbcl3 (s) h = 314 kj sbcl3 (s) cl2 (g) sbcl5 (s) h= 80 kja. -290 KJb. -394 KJc. +394 KJd. -234 KJe. +234 KJ You rent an apartment that costs \$800$800 per month during the first year, but the rent is set to go up 9. 5% per year. What would be the rent of the apartment during the 9th year of living in the apartment? Round to the nearest tenth (if necessary) william's system is locking up during windows boot. he reboots and presses f8 to bring up the boot menu and then selects "enable boot logging" in what file will the results be stored? Determine the confidence level for each of the following large-sample one-sided confidence bounds:a. Upper bound: x+.84snb. Lower bound: x2.05snc. Upper bound: x+.67sn a vertical spring (ignore its mass), whose spring constant is 895 n/m , is attached to a table and is compressed down by 0.150 m . a) What upward speed can it give to a 0.30 kg ball when released?b) How high above its original position (spring compressed) will the ball fly?